Med Surg II Practice Questions

¡Supera tus tareas y exámenes ahora con Quizwiz!

A client is hospitalized for open reduction of a fractured femur. During the postoperative assessment, the nurse notes that the client is restless and she observes petechiae on the client's chest. Which nursing action is indicated first? Elevate the affected extremity. Contact the nursing supervisor. Administer oxygen. Contact the physician.

Administer oxygen.

A patient has questioned the nurse's administration of IV normal saline, asking whether sterile water would be a more appropriate choice than "saltwater." Under what circumstances would the nurse administer electrolyte-free water intravenously? Never, because it rapidly enters red blood cells, causing them to rupture. When the patient is severely dehydrated resulting in neurologic signs and symptoms When the patient is in excess of calcium and/or magnesium ions When a patient's fluid volume deficit is due to acute or chronic renal failure

Never, because it rapidly enters red blood cells, causing them to rupture.

Which of the following is the initial diagnostic in suspected stroke? Noncontrast computed tomography (CT) CT with contrast Magnetic resonance imaging (MRI) Cerebral angiography

Noncontrast computed tomography (CT)

A patient is being treated for a fractured hip and the nurse is aware of the need to implement interventions to prevent muscle wasting and other complications of immobility. What intervention best addresses the patient's need for exercise? Performing gentle leg lifts with both legs Performing massage to stimulate circulation Encouraging frequent use of the overbed trapeze Encouraging the patient to log roll side to side once per hour

Encouraging frequent use of the overbed trapeze

A client has discussed therapy for his HIV-positive status. What does the nurse understand is the goal of antiretroviral therapy? Reverse the HIV+ status to a negative status. Treat mycobacterium avium complex. Eliminate the risk of AIDS. Keep the CD4 cell count above 350/mm3 and viral load undetectable.

Keep the CD4 cell count above 350/mm3 and viral load undetectable.

A client is admitted with weakness, expressive aphasia, and right hemianopia. The brain MRI reveals an infarct. The nurse understands these symptoms to be suggestive of which of the following findings? Transient ischemic attack (TIA) Left-sided cerebrovascular accident (CVA) Right-sided cerebrovascular accident (CVA) Completed Stroke

Left-sided cerebrovascular accident (CVA)

A provider asks the nurse to teach a patient with low back pain how to sit in order to minimize pressure on the spine. Which of the following teaching points would the nurse include? Select all that apply. Sit in a straight-backed chair with arm rests. Use a firm pillow placed behind the thoracic vertebrae to straighten the small of the back. Avoid hip extension. Place feet flat on the floor. Sit with the buttocks "tucked under."

Sit in a straight-backed chair with arm rests. Avoid hip extension. Place feet flat on the floor. Sit with the buttocks "tucked under."

Which of the following statements describes external fixation? The surgeon inserts metal pins into the bone or bones from outside the skin surface and then attaches a compression device to the pins. The surgeon secures the bone with metal screws, plates, rods, nails, or pins. A cast or other mode of immobilization is applied. The bone is restored to its normal position by external manipulation. The bone is surgically exposed and realigned.

The surgeon inserts metal pins into the bone or bones from outside the skin surface and then attaches a compression device to the pins.

A client is hospitalized when they present to the Emergency Department with right-sided weakness. Within 6 hours of being admitted, the neurologic deficits had resolved and the client was back to their presymptomatic state. The nurse caring for the client knows that the probable cause of the neurologic deficit was what? Left-sided stroke Right-sided stroke Cerebral aneurysm Transient ischemic attack

Transient ischemic attack

Which of the following statements reflects the treatment of HIV infection? Treatment should be offered to all patients once they reach CDC category B: HIV symptomatic. Treatment should be offered to only select patients once they reach CDC category B: HIV symptomatic. Treatment should be offered to individuals with plasma HIV RNA levels less than 55,000 copies/mL (RT-PCR assay). Treatment of HIV infection for an individual patient is based on the clinical condition of the patient, CD4 T cell count level, and HIV RNA (viral load).

Treatment of HIV infection for an individual patient is based on the clinical condition of the patient, CD4 T cell count level, and HIV RNA (viral load).

Which of the following patient statements demonstrates an understanding related to Pap smear testing? "I plan to postpone my first Pap test because it is a painful experience." "I have my Pap test every other year to reduce cost related to this expensive screening." "Pap tests are not indicated for older women past childbearing age." "I need to have my first Pap test when I turn 21 or earlier if I become sexually active."

"I need to have my first Pap test when I turn 21 or earlier if I become sexually active."

A client suspected of having systemic lupus erythematosus (SLE) is being scheduled for testing. She asks which of the tests ordered will determine if she is positive for the disorder. Which statement by the nurse is most accurate? "You should discuss that matter with your physician." "The diagnosis won't be based on the findings of a single test but by combining all data found." "SLE is a very serious systemic disorder." "Tell me more about your concerns about this potential diagnosis."

"The diagnosis won't be based on the findings of a single test but by combining all data found."

A client with early stage rheumatoid arthritis asks the nurse what they can do to help ease the symptoms of their disease. What would be the best response by the nurse? "The doctor could prescribe anti-inflammatory drugs." "The doctor could prescribe antipyretic drugs." "The doctor could prescribe antineoplastic drugs." "The doctor could prescribe antihypertensive drugs."

"The doctor could prescribe anti-inflammatory drugs."

A client whose physical findings suggest a hyperpituitary condition undergoes an extensive diagnostic workup. Test results reveal a pituitary tumor, which necessitates a transsphenoidal hypophysectomy. The evening before the surgery, the nurse reviews preoperative and postoperative instructions given to the client earlier. Which postoperative instruction should the nurse emphasize? "You must lie flat for 24 hours after surgery." "You must avoid coughing, sneezing, and blowing your nose." "You must restrict your fluid intake." "You must report ringing in your ears immediately."

"You must avoid coughing, sneezing, and blowing your nose."

A client with rheumatoid arthritis presents to the clinic for a second dose of gold thioglucose (Solganal). The dose prescribed is 25 mg intramuscularly. The medication is available in 50 mg per ml. How many milliliters will the nurse administer to the client?

0.5

Because of Mr. White's recent blood clot, he was given an order for 30,000 units of heparin to be added to 500 mL of normal saline. The order is to infuse the heparin solution via IV at 50 mL/hr using macrodrip tubing (15 gtts/min). How many hours will the IV run? 1.5 hours 6 hours 10 hours 15.5 hours

10 Hours

A client returns from the operating room after receiving extensive abdominal surgery. He has 1,000 mL of lactated Ringer's solution infusing via a central line. The physician orders the IV fluid to be infused at 125 mL/hr plus the total output of the previous hour. The drip factor of the tubing is 15 gtt/min, and the output for the previous hour was 75 mL via Foley catheter, 50 mL via nasogastric tube, and 10 mL via Jackson Pratt tube. For how many drops per mintue should the nurse set the IV flow rate to deliver the correct amount of fluid? Enter the correct number ONLY.

65

The current phase of a patient's treatment for a burn injury prioritizes wound care, nutritional support, and prevention of complications such as infection. Based on these care priorities, the patient is in what phase of burn care? Emergent Immediate resuscitative Acute Rehabilitation

Acute

A client with a history of alcohol abuse comes to the emergency department and complains of abdominal pain. Laboratory studies help confirm a diagnosis of acute pancreatitis. The client's vital signs are stable, but the client's pain is worsening and radiating to his back. Which intervention takes priority for this client? Placing the client in a semi-Fowler's position Maintaining nothing-by-mouth (NPO) status Administering morphine I.V. as ordered Providing mouth care

Administering morphine I.V. as ordered

A patient who was diagnosed with Parkinson's disease several months ago recently began treatment with levodopa-carbidopa. The patient and his family are excited that he has experienced significant symptom relief. The nurse should be aware of what implication of the patient's medication regimen? The patient is in a "honeymoon period" when adverse effects of levodopa-carbidopa are not yet evident. Benefits of levodopa-carbidopa do not peak until 6 to 9 months after the initiation of treatment. The patient's temporary improvement in status is likely unrelated to levodopa-carbidopa. Benefits of levodopa-carbidopa often diminish after 1 or 2 years of treatment.

Benefits of levodopa-carbidopa often diminish after 1 or 2 years of treatment.

A patient returns to the floor after a laparoscopic cholecystectomy. The nurse should assess the patient for signs and symptoms of what serious potential complication of this surgery? Diabetic coma Decubitus ulcer Wound evisceration Bile duct injury

Bile duct injury

Diagnosis of skin cancer is confirmed by which of the following diagnostic tests? Skin scraping Blood culture Visual interpretation Biopsy

Biopsy

A client is suspected of having prostate cancer. The nurse would expect to prepare the client for which of the following to confirm the diagnosis? Digital rectal examination (DRE) Prostate-specific antigen (PSA) testing Transuretheral needle ablation (TUNA) Biopsy via transrectal ultrasound

Biopsy via transrectal ultrasound

A home care nurse assesses for disease complications in a client with bone cancer. Which laboratory value may indicate the presence of a disease complication? Potassium level of 6.3 mEq/L Calcium level of 11.6 mg/dl Sodium level of 110 mEq/L Magnesium level of 0.9 mg/dl

Calcium level of 11.6 mg/dl

A patient diagnosed with a cerebral aneurysm reports a severe headache to the nurse. What action is a priority for the nurse? Sit with the patient for a few minutes. Administer an analgesic. Inform the nurse-manager. Call the physician immediately.

Call the physician immediately

The nurse is collecting assessment data on a client who is reporting a vaginal discharge that is cottage cheese-like in appearance. Which pathogen is the most likely cause for this symptom? Gonococci Candida albicans Trichomonas vaginalis Gardnerella vaginalis

Candida albicans

Which of the following is clinical manifestation of cholelithiasis? Epigastric distress prior to a meal Clay-colored stools Upper left quadrant abdominal pain Nonpalpable abdominal mass

Clay-colored stools

The nurse is evaluating the plan of care for a client with an allergic disorder who has a nursing diagnosis of deficient knowledge related to measures for allergy control. Which of the following would indicate to the nurse that the outcome has been met? Client states the need for coughing and deep breathing. Client demonstrates appropriate coping strategies for dealing with a chronic disorder. Client identifies methods for reducing exposure risk to allergens. Client reports an absence of symptoms associated with the allergy.

Client identifies methods for reducing exposure risk to allergens.

The nurse assesses a patient who experienced a reaction to a bee sting. The patient's clinical findings indicate a pre-shock condition, which is evidenced by: Cold, clammy skin and tachycardia. A systolic blood pressure of 75 mm Hg. A heart rate of 140. Crackles and shallow breathing.

Cold, clammy skin and tachycardia.

A public health nurse is teaching a health promotion workshop that focuses on vision and eye health. What should this nurse cite as the most common causes of blindness and visual impairment among adults over the age of 40? Select all that apply. Diabetic retinopathy Trauma Macular degeneration Cytomegalovirus Glaucoma

Diabetic retinopathy Macular degeneration Glaucoma

The nurse is assessing a patient admitted with renal stones. During the admission assessment, what parameters would be priorities for the nurse to address? Select all that apply. Dietary history Family history of renal stones Medication history Surgical history Vaccination history

Dietary history Family history of renal stones Medication history

A patient who had a Roux-en-Y bypass procedure for morbid obesity ate a chocolate chip cookie after a meal. After ingestion of the cookie, the patient complained of cramping pains, dizziness, and palpitation. After having a bowel movement, the symptoms resolved. What should the patient be educated about regarding this event? Gastric outlet obstruction Dumping syndrome Bile reflux Celiac disease

Dumping syndrome

A nurse is providing care for a patient whose recent colostomy has contributed to a nursing diagnosis of Disturbed Body Image Related to Colostomy. What intervention best addresses this diagnosis? Encourage the patient to conduct online research into colostomies. Engage the patient in the care of the ostomy to the extent that the patient is willing. Emphasize the fact that the colostomy was needed to alleviate a much more serious health problem. Emphasize the fact that the colostomy is temporary measure and is not permanent.

Engage the patient in the care of the ostomy to the extent that the patient is willing.

A client is hospitalized with Guillain-Barré syndrome. Which nursing assessment finding is most significant? Warm, dry skin Urine output of 40 ml/hour Soft, nondistended abdomen Even, unlabored respirations

Even, unlabored respirations

A group of students is reviewing information about the liver and associated disorders. The group demonstrates understanding of the information when they identify which of the following as a primary function of the liver? Breakdown amino acids Convert urea into ammonia Excrete bile Break down coagulation factors

Excrete bile

The nurse in the medical ICU is caring for a patient who is in respiratory acidosis due to inadequate ventilation. What diagnosis could the patient have that could cause inadequate ventilation? Endocarditis Multiple myeloma Guillain-Barré syndrome Overdose of amphetamines

Guillain-Barré syndrome

A client with AIDS is brought to the clinic by their family. The family tells the nurse the client has become forgetful, with a limited attention span, decreased ability to concentrate, and delusional thinking. What condition is represented by these symptoms? Distal sensory polyneuropathy (DSP) Candidiasis HIV encephalopathy Cytomegalovirus (CMV)

HIV encephalopathy

A patient is brought to the emergency department with a burn injury. The nurse knows that the first systemic event after a major burn injury is what? Hemodynamic instability Gastrointestinal hypermotility Respiratory arrest Hypokalemia

Hemodynamic instability

Which term refers to the shifting of brain tissue from an area of high pressure to an area of low pressure? Herniation Autoregulation Cushing's response Monro-Kellie hypothesis

Herniation

The classic lesions of impetigo are manifested as which of the following? Comedones in the facial area Honey crusted lesions on an erythematous base Abscess of skin and subcutaneous tissue Patches of grouped vesicles on red and swollen skin

Honey crusted lesions on an erythematous base

A group of students are reviewing information about osteoporosis in preparation for a class discussion. The students demonstrate a need for additional review when they state which of the following as a risk factor? Excess caffeine intake Prolonged corticosteroid use Hypothyroidism Prolonged immobility

Hypothyroidism

Following a serious thermal burn, which complication will the nurse take action to prevent first? Tissue hypoxia Infection Renal failure Hypovolemia

Hypovolemia

Which nursing diagnosis is least appropriate for the client with rheumatoid arthritis? Imbalanced nutrition: greater than body requirements Deficient knowledge: symptom management Impaired physical mobility Chronic pain

Imbalanced nutrition: greater than body requirements

A child tips a pot of boiling water onto his bare legs. The mother should: Avoid touching the burned skin and take the child to the nearest emergency department. Cover the child's legs with ice cubes secured with a towel. Immerse the child's legs in cool water. Liberally apply butter or shortening to the burned areas.

Immerse the child's legs in cool water.

Which of the following skin conditions is caused by staphylococci, streptococci, or multiple bacteria? Scabies Pediculosis capitis Poison ivy Impetigo

Impetigo

When obtaining the vital signs of a client with multiple traumatic injuries, a nurse detects bradycardia, bradypnea, and systolic hypertension. The nurse must notify the physician immediately because these findings may reflect which complication? Shock Encephalitis Increased intracranial pressure (ICP) Status epilepticus

Increased intracranial pressure (ICP)

A patient's medication regimen for the treatment of hypertension includes hydrochlorothiazide. Following administration of this medication, the nurse should anticipate what effect? Drowsiness or lethargy Increased urine output Decreased heart rate Mild agitation

Increased urine output

A public health nurse has been asked to provide a health promotion session for men at a wellness center. What should the nurse inform the participants about testicular cancer? It is most common among men over 55. It is one of the least curable solid tumors. It typically does not metastasize. It is highly responsive to treatment.

It is highly responsive to treatment.

Assessment of a client reveals signs and symptoms of Paget's disease. Which of the following would be most likely? Skull narrowing Lordosis Long bone bowing Waddling gait

Long bone bowing

A nurse in an ophthalmology practice is obtaining a history from a new client with glaucoma. The nurse asks the client if she understands her new diagnosis. The client responds by saying she has had time to read about the diagnosis and understands that the type of glaucoma she has is due to the degeneration and obstruction of the trabecular meshwork, whose original function is to absorb the aqueous humor. The loss of absorption will lead to an increased resistance, and thus a chronic, painless buildup of pressure in the eye. Which type of glaucoma has the client described? Open angle Angle closure Congenital Secondary

Open angle

A patient with spinal cord injury is ready to be discharged home. A family member asks the nurse to review potential complications one more time. What are the potential complications that should be monitored for in this patient? Select all that apply. Orthostatic hypotension Autonomic dysreflexia DVT Salt-wasting syndrome Increased ICP

Orthostatic hypotension Autonomic dysreflexia DVT

A client is informed that he has a benign bone tumor but is the type of tumor that may become malignant. What type of tumor does the nurse know that this is characteristic of? Osteochondroma Enchondroma Osteoclastoma Osteoid osteoma

Osteoclastoma

A nurse is caring for a patient who is recovering in the hospital following orthopedic surgery. The nurse is performing frequent assessments for signs and symptoms of infection in the knowledge that the patient faces a high risk of what infectious complication? Cellulitis Septic arthritis Sepsis Osteomyelitis

Osteomyelitis

Which finding is an early indicator of bladder cancer? Painless hematuria Occasional polyuria Nocturia Dysuria

Painless hematuria

Fat emboli are a major cause of death for patients with fractures. What are the significant signs and symptoms? Select all that apply. Petechiae possibly due to a transient thrombocytopenia Substernal chest pain Bradycardia Hypoxia Bradypnea

Petechiae possibly due to a transient thrombocytopenia Substernal chest pain Hypoxia

After a transsphenoidal adenohypophysectomy, a client is likely to undergo hormone replacement therapy. A transsphenoidal adenohypophysectomy is performed to treat which type of cancer? Esophageal carcinoma Pituitary carcinoma Laryngeal carcinoma Colorectal carcinoma

Pituitary carcinoma

A 35-year-old client has had very scanty, infrequent menses since their onset. She is intent on discovering the reason for her inability to conceive. She also has a history of type 2 diabetes and acne. What could be the reason for her menstrual disorder? Polycystic ovarian syndrome Premature ovarian failure Cystocele All options are correct.

Polycystic ovarian syndrome

Which well-recognized sign of meningitis is exhibited when the patient's neck is flexed and flexion of the knees and hips is produced? Positive Kerning's sign Photophobia Positive Brudzinski sign Nuchal rigidity

Positive Brudzinski sign

A night-shift nurse receives a call from the emergency department about a client with herpes zoster who is going to be admitted to the floor. Based on this diagnosis, where should the nurse assign the client? Semi-private room with a client diagnosed with pneumonia Semi-private room with a client who had chickenpox and was admitted with a GI bleed Private room Isolation room with negative airflow

Private room

A nurse is planning care for a client who experienced a stroke in the right hemisphere of his brain. What should the nurse do? Anticipate the client will exhibit some degree of expressive or receptive aphasia. Place the wheelchair on the client's left side when transferring him into a wheelchair. Provide close supervision because of the client's impulsiveness and poor judgment. Support the right arm with a sling or pillow to prevent subluxation.

Provide close supervision because of the client's impulsiveness and poor judgment.

What is your assessment of this skin finding? Psoriasis Impetigo Basal Cell Carcinoma Thermal burn injury

Psoriasis

Which of the following is the cause of shingles? Parasitic fungi Itch mite Reactivated virus Hormonal change

Reactivated virus

A 27-year-old female patient is diagnosed with invasive cervical cancer and is told she needs to have a hysterectomy. One of the nursing diagnoses for this patient is "disturbed body image related to perception of femininity." What intervention would be most appropriate for this patient? Reassure the patient that she will still be able to have children. Reassure the patient that she does not have to have sex to be feminine. Reassure the patient that you know how she is feeling and that you feel her anxiety and pain. Reassure the patient that she will still be able to have intercourse with sexual satisfaction and orgasm.

Reassure the patient that she will still be able to have intercourse with sexual satisfaction and orgasm.

A patient diagnosed with osteoporosis is being discharged home. Which of the following is the priority education the nurse should provide? Removing all small rugs from the home Participating in weight-bearing exercises Classifying medications Increasing calcium and vitamin D in the diet

Removing all small rugs from the home

Which of the following is included as a dietary guideline for a patient who has had bariatric surgery? Restrict total meal size to less than 1 cup. Make sure to drink liquids while eating. Limit soda to 1 can per day. Eat four small meals a day.

Restrict total meal size to less than 1 cup.

Which of the following are clinical manifestations associated with increased intracranial pressure (ICP)? Select all that apply. Seizures Nausea with or without vomiting Papilledema Headache Angina

Seizures Nausea with or without vomiting Papilledema Headache

A client is admitted with nausea, vomiting, and diarrhea. His blood pressure on admission is 74/30 mm Hg. The client is oliguric and his blood urea nitrogen (BUN) and creatinine levels are elevated. The physician will most likely write an order for which treatment? Encourage oral fluids. Administer furosemide (Lasix) 20 mg I.V. Start hemodialysis after a temporary access is obtained. Start I.V. fluids with a normal saline solution bolus followed by a maintenance dose.

Start I.V. fluids with a normal saline solution bolus followed by a maintenance dose.

Which of the following is one of the most important prognostic factor in breast cancer? Obesity Status of lymph nodes Age of patient Family history

Status of lymph nodes

A patient is exploring treatment options after being diagnosed with age-related cataracts that affect her vision. What treatment is most likely to be used in this patient's care? Antioxidant supplements, vitamin C and E, beta-carotene, and selenium Eyeglasses or magnifying lenses Corticosteroid eye drops Surgical intervention

Surgical intervention

A nurse is monitoring a client with peptic ulcer disease. Which of the following assessment findings would most likely indicate perforation of the ulcer? Choose all that apply. Tachycardia Hypotension Mild epigastric pain A rigid, board-like abdomen Diarrhea

Tachycardia Hypotension A rigid, board-like abdomen

The nurse is preparing health education for a patient who is being discharged after hospitalization for a hemorrhagic stroke. What content should the nurse include in this education? Mild, intermittent seizures can be expected. Take ibuprofen for complaints of a serious headache. Take antihypertensive medication as ordered. Drowsiness is normal for the first week after discharge.

Take antihypertensive medication as ordered.

A client's left leg is in skeletal traction with a Thomas leg splint and Pearson attachment. Which intervention should the nurse include in this client's care plan? Apply the traction straps snugly. Assess the client's level of consciousness. Remove the traction at least every 8 hours. Teach the client how to prevent problems caused by immobility.

Teach the client how to prevent problems caused by immobility.

A client's left leg is in skeletal traction with a Thomas leg splint and Pearson attachment. Which intervention should the nurse include in this client's care plan? Apply the traction straps snugly. Assess the client's level of consciousness. Remove the traction at least every 8 hours. Teach the client how to prevent problems caused by immobility.

Teach the client how to prevent problems caused by immobility.

The client arrives at a public health clinic worried that she has breast cancer since finding a lump in her breast. When assessing the breast, which assessment finding is characteristic of fibrocystic disease? One breast is larger than the other. The lump is firm and nonmovable. The lump is round and movable. Nipple retractions are noted.

The lump is round and movable.

T-cell deficiency occurs when which of the following glands fails to develop normally during embryogenesis? Thyroid Thymus Pituitary Adrenal

Thymus

Which of the following are associated with compartment syndrome? Select all that apply. Trauma from accidents Surgery Casts Tight bandages Crushing injuries

Trauma from accidents Surgery Casts Tight bandages Crushing injuries

A patient has developed hepatic encephalopathy secondary to cirrhosis and is receiving care on the medical unit. The patient's current medication regimen includes lactulose (Cephulac) four times daily. What desired outcome should the nurse relate to this pharmacologic intervention? Two to 3 soft bowel movements daily Significant increase in appetite and food intake Absence of nausea and vomiting Absence of blood or mucus in stool

Two to 3 soft bowel movements daily

Which of the following medications are used to decrease portal pressure, halting bleeding of esophageal varices? Spironolactone (Aldactone) Vasopressin (Pitressin) Nitroglycerin Cimetidine (Tagamet)

Vasopressin (Pitressin) Vasopressin may be the initial therapy for esophageal varices, because it produces constriction of the splanchnic arterial bed and decreases portal hypertension.

A patient falls while skiing and sustains a supracondylar fracture. What does the nurse know is the most serious complication of a supracondylar fracture of the humerus? Hemarthrosis Paresthesia Malunion Volkmann's ischemic contracture

Volkmann's ischemic contracture

A patient is diagnosed with amyotrophic lateral sclerosis, also known as ALS or Lou Gehrig's disease. The nurse understands that the symptoms of the disease will begin in what way? Ascending paralysis Numbness and tingling in the lower extremities Weakness starting in the muscles supplied by the cranial nerves Jerky, uncontrolled movements in the extremities

Weakness starting in the muscles supplied by the cranial nerves

Which nursing action is most appropriate for a client hospitalized with acute pancreatitis? Withholding all oral intake, as ordered, to decrease pancreatic secretions Administering meperedine, as ordered, to relieve severe pain Limiting I.V. fluids, as ordered, to decrease cardiac workload Keeping the client supine to increase comfort

Withholding all oral intake, as ordered, to decrease pancreatic secretions

The digestion of carbohydrates is aided by lipase. amylase. trypsin. secretin.

amylase.

A client has just been diagnosed with early glaucoma. During a teaching session, the nurse should: provide instructions on eye patching. assess the client's visual acuity. demonstrate eyedrop instillation. teach about intraocular lens cleaning.

demonstrate eyedrop instillation.

A nurse is monitoring a client for increasing intracranial pressure (ICP). Early signs of increased ICP include: pupillary changes. diminished responsiveness. decreasing blood pressure. elevated temperature.

diminished responsiveness

To help minimize calcium loss from a hospitalized client's bones, the nurse should: reposition the client every 2 hours. encourage the client to walk in the hall. provide the client dairy products at frequent intervals. provide supplemental feedings between meals.

encourage the client to walk in the hall

To help minimize calcium loss from a hospitalized client's bones, the nurse should: reposition the client every 2 hours. encourage the client to walk in the hall. provide the client dairy products at frequent intervals. provide supplemental feedings between meals.

encourage the client to walk in the hall.

A physician has ordered a wet-to-damp dressing for an infected pressure ulcer. The nurse knows that the primary reason for this treatment is to: prevent the spread of the infection. debride the wound. keep the wound moist. reduce pain.

keep the wound moist.

When obtaining the health history from a client with retinal detachment, a nurse expects the client to report: light flashes and floaters in front of the eye. a recent driving accident while changing lanes. headaches, nausea, and redness of the eyes. frequent episodes of double vision.

light flashes and floaters in front of the eye.

A patient with increased intracranial pressure has a cerebral perfusion pressure (CPP) of 40 mm Hg. This CPP reading is considered high. low. within normal limits. inaccurate.

low

A client is admitted for treatment of glomerulonephritis. On initial assessment, the nurse detects one of the classic signs of acute glomerulonephritis of sudden onset. Such signs include: periorbital edema. green-tinged urine. moderate to severe hypotension. polyuria.

periorbital edema.

A client comes to the physician's office for treatment of severe sunburn. The nurse takes this opportunity to discuss the importance of protecting the skin from the sun's damaging rays. Which instruction best prevents skin damage? "Minimize sun exposure from 1 to 4 p.m., when the sun is strongest." "Use a sunscreen with a sun protection factor of 6 or higher." "Apply sunscreen even on overcast days." "When at the beach, sit in the shade to prevent sunburn."

"Apply sunscreen even on overcast days."

A patient with stage II breast cancer has received her treatment plan that includes radiation therapy. The nurse knows that the patient understands radiation treatment when she makes which of the following statements? "Radiation therapy will reduce the chance of breast cancer recurrence." "Radiation and a modified radical mastectomy will cure my breast cancer." "I will receive my first radiation treatment before I leave the hospital." "I will plan on radiation treatments once a week for 5 weeks."

"Radiation therapy will reduce the chance of breast cancer recurrence."

Convert the following quantity to the equivalent unit indicated: 250 mcg = ______________ mg 0.25 mg 250 mg 2.5 mg 0.025 mg

0.25 mg

A patient is admitted to the emergency department (ED) following a motorcycle accident. Upon assessment, the patient's vital signs reveal blood pressure (BP) of 80/60 mm Hg and heart rate (HR) of 145 beats per minute (bpm). The patient's skin is cool and clammy. Which of the following patient medical orders will the nurse complete first? Two large-bore IVs and begin crystalloid fluids 100% oxygen per nonrebreather mask C-spine x-rays Type and cross match

100% oxygen per nonrebreather mask

Perforation of the appendix generally occurs within which timeframe of the onset of pain if no intervention is done? 24 hours 12 hours 36 hour 48 hours

24 hours

A client is brought to the emergency department with partial-thickness and full-thickness burns on the left arm, left anterior leg, and anterior trunk. Using the Rule of Nines, what is the total body surface area that has been burned? 18% 27% 30% 36%

36%

The nurse is administering eye drops to a patient with glaucoma. After instilling the patient's first medication, how long should the nurse wait before instilling the patient's second medication into the same eye? 30 seconds 1 minute 3 minutes 5 minutes

5 minutes

The nurse has been asked to explain the cause of angina pain to a patient's family. Choose the best statement. The pain is due to: Incomplete blockage of a major coronary artery. A destroyed part of the heart muscle. Complete closure of an artery. A lack of oxygen in the heart muscle that causes the death of cells.

A lack of oxygen in the heart muscle that causes the death of cells.

The nurse is caring for a patient who is being assessed for brain death. Which of the following are cardinal signs of brain death? Select all that apply. Absence of brain stem reflexes No brain waves Apnea Coma

Absence of brain stem reflexes Apnea Coma

The nurse is caring for a patient with permanent neurologic impairments resulting from a traumatic head injury. When working with this patient and family, what mutual goal should be prioritized? Achieve as high a level of function as possible. Enhance the quantity of the patient's life. Teach the family proper care of the patient. Provide community assistance.

Achieve as high a level of function as possible.

What is the most common cause of small-bowel obstruction? Hernias Neoplasms Adhesions Volvulus

Adhesions

Which of the following is a reason why ovarian cancer is largely considered to be a lethal cancer of the female reproductive system? All of the above Tumors present with nonspecific symptoms There is no effective screening test. Tumors are typically far advanced and inoperable by the time they are diagnosed.

All of the above

A 51-year-old client is being seen by an urologist for perineal pain, low back pain, fever lasting 5 days, and painful urination. The physician confirms a diagnosis of prostatitis and orders treatment. During your client education session, which of the following recommendations do you make? All options are correct. Avoid caffeine. Regularly drain the gland. Complete the prescribed antibiotic treatment.

All options are correct.

An 80-year-old client is being seen by the urologist because of erectile dysfunction. This client has a history of hypertension and heart disease. Which of the following would be the most likely cause of his erectile dysfunction? All options are correct. Hypertension medication Atherosclerosis Depression

All options are correct.

The single modality of pharmacologic therapy for chronic type B viral hepatitis is: Alpha-interferon Hepsera Epivir Baraclude

Alpha-interferon

A client with a history of chronic renal failure receives hemodialysis treatments three times per week through an arteriovenous (AV) fistula in the left arm. Which intervention should the nurse include in the care plan? Keep the AV fistula site dry. Keep the AV fistula wrapped in gauze. Take the client's blood pressure in the left arm. Assess the AV fistula for a bruit and thrill.

Assess the AV fistula for a bruit and thrill.

A nurse is performing a home visit to a patient who is recovering following a long course of inpatient treatment for burn injuries. When performing this home visit, the nurse should do which of the following? Assess the patient for signs of electrolyte imbalances. Administer fluids as ordered. Assess the risk for injury recurrence. Assess the patient's psychosocial state.

Assess the patient's psychosocial state

The nurse has implemented a bladder retraining program for an older adult patient. The nurse places the patient on a timed voiding schedule and performs an ultrasonic bladder scan after each void. The nurse notes that the patient typically has approximately 50 mL of urine remaining in her bladder after voiding. What would be the nurse's best response to this finding? Perform a straight catheterization on this patient. Avoid further interventions at this time, as this is an acceptable finding. Place an indwelling urinary catheter. Press on the patient's bladder in an attempt to encourage complete emptying.

Avoid further interventions at this time, as this is an acceptable finding.

The nurse instructs the client with gastroesophageal reflux disease (GERD) regarding dietary measures. The client has understood the recommended dietary changes by: Eliminating spicy foods. Avoiding chocolate and coffee. Eliminating cucumbers and other foods with seeds. Avoiding steamed foods.

Avoiding chocolate and coffee.

Which of the following is an example of a direct measurement technique for evaluation of the teaching--learning process? Behavioral observation Patient satisfaction surveys Attitude surveys Instruments that evaluate specific health status variables

Behavioral observation

A nurse is providing an educational class to a group of older adults at a community senior center. In an effort to prevent osteoporosis, the nurse should encourage participants to ensure that they consume the recommended adequate intake of what nutrients? Select all that apply. Vitamin B12 Potassium Calcitonin Calcium Vitamin D

Calcium Vitamin D

The nurse is to check residual urine amounts for a client experiencing urinary retention. Which of the following would be most important? Set up a routine schedule of every 4 hours to check for residual urine. Check for residual after the client reports the urge to void. Record the volume of urine obtained. Catheterize the client immediately after the client voids.

Catheterize the client immediately after the client voids.

A nurse is assisting with a neurological examination of a client who reports a headache in the occipital area and shows signs of ataxia and nystagmus. Which of the following conditions is the most likely reason for the client's problems? Frontal lobe abscess Temporal lobe abscess Cerebellar abscess Wernicke's abscess

Cerebellar abscess

A patient presents to the emergency room with complaints of having an "exploding headache" for the last 2 hours. The patient is immediately seen by a triage nurse who suspects the patient is experiencing a stroke. Which of the following is a possible cause based on the characteristic symptom? Large artery thrombosis Cerebral aneurysm Cardiogenic emboli Small artery thrombosis

Cerebral aneurysm

A woman in her late thirties has been having unusually heavy menstrual periods combined with occasional urine and stool leakage over the last few weeks. On further enquiry, she reveals that she also has postcoital pain and bleeding. To which of the following diagnoses do you think the investigations are most likely to lead? Hodgkin's disease Cervical cancer Colorectal cancer Cancer of the urinary tract

Cervical cancer

A nurse suspects that a client has Huntington disease based on which assessment finding? Slurred speech Disorganized gait Chorea Dementia

Chorea

A client is admitted to an acute care facility after an episode of status epilepticus. After the client is stabilized, which factor is most beneficial in determining the potential cause of the episode? The type of anticonvulsant prescribed to manage the epileptic condition Recent stress level Recent weight gain and loss Compliance with the prescribed medication regimen

Compliance with the prescribed medication regimen

A client diagnosed with polycythemia vera has come into the clinic because he has developed a nighttime cough, fatigue, and shortness of breath. What complication would you suspect in this client? Stroke Tissue infarction Congestive heart failure Pulmonary embolus

Congestive heart failure

A client with gastric cancer is having a resection. What is the nursing management priority for this client? Discharge planning Correcting nutritional deficits Preventing deep vein thrombosis (DVT) Teaching about radiation treatment

Correcting nutritional deficits

Which of the following are exclusions for bariatric surgery. Select all that apply. Current drug use Reversible endocrine disorder that can cause obesity Uncontrolled, severe psychiatric illness Alcohol abuse Adolescent

Current drug use Reversible endocrine disorder that can cause obesity Uncontrolled, severe psychiatric illness Alcohol abuse

Which of the following is a downward displacement of the bladder toward the vaginal orifice? Rectocele Cystocele Vulvodynia Fistula

Cystocele

Decerebrate Decorticate

Decerebrate Decorticate

Nursing assessment of hearing loss in an elderly patient includes evaluation of age-related changes, as well as a history of current illnesses and medications. Which of the following factors are associated with ototoxic effects? Select all that apply. Coronary artery disease Diabetes mellitus Loop diuretics (e.g., Lasix) Asthma Bacterial meningitis Gentamicin

Diabetes mellitus Loop diuretics (e.g., Lasix) Bacterial meningitis Gentamicin

A patient receiving vasopressin for the management of active bleeding due to esophageal varices should be assessed for evidence of the drug's most serious complication. Therefore, the nurse should frequently check the patient's: Urinary output. Electrocardiogram. Electrolytes. Liver enzymes.

Electrocardiogram.

Which of the following is the most common clinical manifestation of multiple sclerosis? Pain Fatigue Spasticity Ataxia

Fatigue

A clinic nurse is caring for a patient newly diagnosed with fibromyalgia. When developing a care plan for this patient, what would be a priority nursing diagnosis for this patient? Impaired Urinary Elimination Related to Neuropathy Altered Nutrition Related to Impaired Absorption Disturbed Sleep Pattern Related to CNS Stimulation Fatigue Related to Pain

Fatigue Related to Pain

A 32-year-old client is concerned with the lumps that have developed in her breasts and is fearful of cancer. She reports variability in the size of the lumps. What would you expect the physician to indicate is her condition? Fibrocystic breast disease Lactating mastitis Breast cancer All options are correct.

Fibrocystic breast disease

How would you interpret this EKG strip and what nursing interventions would you anticipate? First Degree Heart Block, assess for symptoms, monitor if asymptomatic, refer to cardiologist Normal Sinus Rhythm, no interventions necessary Premature Atrial Contractions (PAC's), record and monitor if asymptomatic Third Degree Heart Block, anticipate pace maker insertion

First Degree Heart Block, assess for symptoms, monitor if asymptomatic, refer to cardiologist

A patient has been admitted to a burn intensive care unit with extensive full-thickness burns over 25% of the body. After ensuring cardiopulmonary stability, what would be the nurse's immediate, priority concern when planning this patient's care? Fluid status Risk of infection Nutritional status Psychosocial coping

Fluid Status

A patient has been brought to the emergency department by paramedics after being found unconscious. The patient's Medic Alert bracelet indicates that the patient has type 1 diabetes and the patient's blood glucose is 22 mg/dL (1.2 mmol/L). The nurse should anticipate what intervention? IV administration of 50% dextrose in water Subcutaneous administration of 10 units of Humalog Subcutaneous administration of 12 to 15 units of regular insulin IV bolus of 5% dextrose in 0.45% NaCl

IV administration of 50% dextrose in water

A nurse educator is teaching a group of recent nursing graduates about their occupational risks for contracting hepatitis B. What preventative measures should the educator promote? Select all that apply. Immunization Use of standard precautions Consumption of a vitamin-rich diet Annual vitamin K injections Annual vitamin B12 injections

Immunization Use of standard precautions

A nurse caring for a patient with a neurogenic bladder knows to assess for the major complication of: Permanent distention Infection Consistent pain Daily and painful spasms

Infection

A nurse is preparing a presentation about cervical cancer for a local woman's group. Which of the following would the nurse include as a possible risk factor for the development of cervical cancer? Sexual activity beginning at a later age Having a single sexual partner Infection with human papilloma virus (HPV) Never having been pregnant

Infection with human papilloma virus (HPV)

A patient has been diagnosed with a frontal lobe brain abscess. Which of the following nursing interventions is appropriate? Assess for facial weakness. Initiate seizure precautions. Assess visual acuity. Ensure that patient takes nothing by mouth (NPO).

Initiate seizure precautions.

A client complains of vertigo. The nurse anticipates that the client may have a problem with which portion of the ear? External ear Middle ear Inner ear Tympanic membrane

Inner ear

An ophthalmologist tells a patient that he has a cataract. The nurse explains to the patient that this means there is: Distortion and loss of central vision. A tendency for the retina to tear. Interference with focusing of a sharp image. Increased corneal exposure.

Interference with focusing of a sharp image.

Which of the following site is the source of most microbes leading to bacterial infection? Intestinal tract Respiratory tract Skin Mucous membranes

Intestinal tract

A nurse practitioner provides health teaching to a patient who has difficulty managing hypertension. This patient is at an increased risk of which type of stroke? Intracerebral hemorrhage Subarachnoid hemorrhage Hemorrhage due to an aneurysm Arteriovenous malformation

Intracerebral hemorrhage

Which of the following aid in diagnosing the risk of anaphylaxis? Intradermal testing Nasal smear Punch biopsy Peripheral blood smears

Intradermal testing

A client has partial-thickness burns on both lower extremities and portions of the trunk. Which I.V. fluid does the nurse plan to administer first? Albumin Dextrose 5% in water (D5W) Lactated Ringer's solution Normal saline solution with 20 mEq of potassium per 1,000 ml

Lactated Ringer's solution

Which of the following are functions of saliva? Select all that apply. Lubrication Protection against harmful bacteria Digestion Elimination Metabolism

Lubrication Protection against harmful bacteria Digestion

The nurse is participating in the care of a patient with increased ICP. What diagnostic test is contraindicated in this patient's treatment? Computed tomography (CT) scan Lumbar puncture Magnetic resonance imaging (MRI) Venous Doppler studies

Lumbar puncture

Which of the following is a brain tumor arising from the supporting structures? Meningiomas Astrocytomas Medulloblastoma Glioblastoma multiforme

Meningiomas

A 45-year-old woman comes into the health clinic for her annual check-up. She mentions to the nurse that she has noticed dimpling of the right breast that has occurred in a few months. What assessment would be most appropriate for the nurse to make? Evaluate the patient's milk production. Palpate the area for a breast mass. Assess the patient's knowledge of breast cancer. Assure the patient that this likely an age-related change.

Palpate the area for a breast mass.

Which of the following is the major cause of morbidity and mortality in patients with acute pancreatitis? Shock Pancreatic necrosis MODS Tetany

Pancreatic necrosis

Which condition is most likely to have a nursing diagnosis of fluid volume deficit? Appendicitis Pancreatitis Cholecystitis Gastric ulcer

Pancreatitis

Which terms refers to the progressive hearing loss associated with aging? Presbycusis Exostoses Otalgia Sensorineural hearing loss

Presbycusis

Stephen Oswald, a 68-year-old retired salesman, was brought by squad into the acute care facility where you practice nursing. His wife accompanies him and relates how Stephen reported a severe headache, shortly after he was unable to talk or move his right arm and leg. His wife indicates Mr. Oswald has hypertension. What should be your focus of management during this phase? Preventing further neurologic damage Reporting changes to the physician Destabilizing client's condition Frequently assessing vital signs

Preventing further neurologic damage

Which action by the nurse would be inappropriate for the client following casting? Protect the cast by covering with a sheet. Handle the cast with the palms of hands. Circulate room air with a portable fan. Petal and smooth the edges of the cast.

Protect the cast by covering with a sheet.

Which instruction is the most important to give a client who has recently had a skin graft? Continue physical therapy. Protect the graft from direct sunlight. Use cosmetic camouflage techniques. Apply lubricating lotion to the graft site.

Protect the graft from direct sunlight.

A patient who was burned in a workplace accident has completed the acute phase of treatment and the plan of care has been altered to prioritize rehabilitation. What nursing action should be prioritized during this phase of treatment? Monitoring fluid and electrolyte imbalances Providing education to the patient and family Treating infection Promoting thermoregulation

Providing education to the patient and family

Treatment of melanoma includes which of the following? Cryosurgery Radical excision Radiation therapy Laser surgery

Radical excision

A nurse formulates a nursing diagnosis of Impaired physical mobility for a client with full-thickness burns on the lower portions of both legs. To complete the nursing diagnosis statement, the nurse should add which "related-to" phrase? Related to fat emboli Related to infection Related to femoral artery occlusion Related to circumferential eschar

Related to circumferential eschar

The hospice nurse is assigned to care for a patient with metastatic bone cancer who wants to remain at home. What is the therapeutic goal in the care of this patient? Prevent the patient from having to go to the hospital for care. Relieve pain and discomfort while promoting quality of life. Increase the activity level of the patient to prevent complications related to immobility. Ensure that the family accepts the patient's imminent death.

Relieve pain and discomfort while promoting quality of life.

A client with Paget's disease comes to the hospital and complains of difficulty urinating. The emergency department physician consults urology. What should the nurse suspect is the most likely cause of the client's urination problem? Renal calculi Urinary tract infection (UTI) Benign prostatic hyperplasia Dehydration

Renal calculi

A client is admitted to the hospital for a fracture of the right femur. Which clinical manifestation supports the diagnosis? Swelling of the right leg Pain in the right thigh Hematoma over the right trochanter Right leg shorter than left

Right leg shorter than left

Which of the following statements correctly matches the nursing diagnosis with its appropriate nursing intervention for a patient suffering with vertigo? Select all that apply. Risk for deficient fluid volume intake: Encourage oral fluids. Powerlessness: Provide a daily schedule of activities. Risk for trauma: Assist with ambulation. Anxiety: Provide information about vertigo and its treatment. Risk for injury: Encourage vestibular therapy.

Risk for deficient fluid volume intake: Encourage oral fluids. Risk for trauma: Assist with ambulation. Anxiety: Provide information about vertigo and its treatment. Risk for injury: Encourage vestibular therapy.

A client with renal failure is undergoing continuous ambulatory peritoneal dialysis. Which nursing diagnosis is the most appropriate for this client? Impaired urinary elimination Toileting self-care deficit Risk for infection Activity intolerance

Risk for infection

When developing the plan of care for a client with a primary immunodeficiency, which nursing diagnosis would be the priority? Anxiety related to an inherited disorder Grieving related to the poor prognosis of the condition Risk for infection related to altered immune cell function Impaired skin integrity related to persistent deep skin abscesses

Risk for infection related to altered immune cell function

The nurse is reviewing the chart of a client with swallowing problems. Which of the following factors would raise suspicion that the client has cancer of the esophagus? Choose all that apply. Smoking history of 20 years Male gender Previous treatment for gastroesophageal reflux disease Caucasian race Age 72 years

Smoking history of 20 years Male gender Previous treatment for gastroesophageal reflux disease

Gynecomastia is a common side effect of which of the following diuretics? Spironolactone (Aldactone) Furosemide (Lasix) Vasopressin (Pitressin) Nitroglycerin (IV)

Spironolactone (Aldactone)

Which of the following devices is designed specifically to support and immobilize a body part in a desired position? Brace Continuous passive motion (CPM) device Splint Trapeze

Splint

A 34-year-old client was playing a pick-up game of basketball when he became injured. He was brought to the urgent care facility where you practice nursing with an extremely painful elbow, which was very edematous. What type of injury would you suspect that the client experienced? Sprain Strain Contusion All options are correct.

Sprain

A nurse is assessing a client and finds a malignant lesion on the client's face. The nurse suspects this is what type of skin cancer? Basal cell carcinoma Squamous cell carcinoma Malignant melanoma Dermatofibroma

Squamous cell carcinoma

Thirty minutes after the nurse begins an intravenous immunoglobulin (IVIG) infusion, the patient complains of itching at the site and a lump in the throat. Which is the first action the nurse should take? Apply a tourniquet above the infusion site. Notify the physician. Stop the infusion. Administer subcutaneous epinephrine.

Stop the infusion.

The nurse is gathering data from laboratory studies for a client who has HIV. The clients T4-cell count is 200/mm³, and the client has been diagnosed with Pneumocystis pneumonia. What does this indicate to the nurse? The client has converted from HIV infection to AIDS. The client has advanced HIV infection. The client's T4-cell count has decreased due to the Pneumocystis pneumonia. The client has another infection present that is causing a decrease in the T4-cell count.

The client has converted from HIV infection to AIDS.

A patient comes to the clinic with some hearing loss. The physician is unable to observe the tympanic membrane due to the accumulation of cerumen. What intervention can the nurse provide so that observation can be made? The nurse can remove the wax with a cerumen curette. The ear can be irrigated with cool water until all of the wax is removed. The nurse can instill a small amount of mineral oil into the canal and have the patient return for removal of the wax. The nurse can instill mineral oil into the canal and immediately irrigate to remove the adherent wax.

The nurse can instill a small amount of mineral oil into the canal and have the patient return for removal of the wax.

A nurse on the neurologic unit is providing care for a patient who has spinal cord injury at the level of C4. When planning the patient's care, what aspect of the patient's neurologic and functional status should the nurse consider? The patient will be unable to use a wheelchair. The patient will be unable to swallow food. The patient will be continent of urine, but incontinent of bowel. The patient will require full assistance for all aspects of elimination.

The patient will require full assistance for all aspects of elimination.

When performing the Weber test, where would the nurse place the vibrating tuning fork? Identify the area on the accompanying figure.

Top of forehead

Nursing students are reviewing information about Parkinson's disease in preparation for class the next day. The students demonstrate understanding of the material when they identify which of the following as a cardinal sign of this disorder? Select all that apply. Tremor Rigidity Bradykinesia Postural instability Intellectual decline

Tremor Rigidity Bradykinesia Postural instability

A severely cognitively impaired adult has had a surgical procedure, and the nurse is having a difficult time assessing the level of pain the client is having postoperatively. What method can the nurse use to obtain data about the client's pain? Have the client point to a smiley face or a frown. Ask the client to point to a pain level between 0 and 10 on a chart. Use behavioral comparison of the client's current and previous behavior patterns. Ask the client loudly if he is having any pain and what level it is.

Use behavioral comparison of the client's current and previous behavior patterns.

When the area of burn is irregular in shape and is scattered over multiple areas of the body, which is the best method for the nurse to obtain a quick assessment of the total body surface area of the burn? Rule of nines Use client's palm size Parkland formula Lund and Browder burns assessment

Use client's palm size

Which assessment finding would create the greatest risk to a client ordered a phosphodiesterase (PDE5) inhibitor such as sildenafil (Viagra)? Use of nitrates History of hypertension Type 2 diabetes Use of diuretics

Use of nitrates

Because of difficulties with hemodialysis, peritoneal dialysis is initiated to treat a client's uremia. Which finding during this procedure signals a significant problem? Blood glucose level of 200 mg/dl White blood cell (WBC) count of 20,000/mm3 Potassium level of 3.5 mEq/L Hematocrit (HCT) of 35%

White blood cell (WBC) count of 20,000/mm3

Which of the following terms refer to the failure to recognize familiar objects perceived by the senses? Agnosia Agraphia Apraxia Perseveration

agnosia

A client has experienced an ischemic stroke that has damaged the temporal (lateral and superior portions) lobe. Which of the following deficits would the nurse expect during assessment of this client? Limited attention span and forgetfulness Hemiplegia or hemiparesis Lack of deep tendon reflexes Auditory agnosia

auditory agnosia

A client develops acute renal failure (ARF) after receiving I.V. therapy with a nephrotoxic antibiotic. Because the client's 24-hour urine output totals 240 ml, the nurse suspects that the client is at risk for: cardiac arrhythmia. paresthesia. dehydration. pruritus.

cardiac arrhythmia.

While assessing the skin of a 45-year-old, fair-skinned female client, the nurse notes a lesion on the medial aspect of her lower leg. It has irregular borders, with various shades of black and brown. The client states that the lesion itches occasionally and bled slightly a few weeks ago. She also reveals a history of sunburns. Based on these signs and symptoms, the nurse suspects: squamous cell carcinoma. actinic keratoses. melanoma. basal cell carcinoma.

melanoma.

The nurse is providing discharge instructions for a slightly overweight client seen in the Emergency Department for chest pain. The client was diagnosed as having gastroesophageal reflux disease. The nurse notes in the client's record that the client is taking carbidopa/levodopa (Sinemet). The nurse questions the physician's order for a low-fat diet elevation of upper body on pillows pantaprazole metoclopramide

metoclopramide

The basic difference between nursing diagnoses and collaborative problems is that collaborative problems can be managed by independent nursing interventions. nursing diagnoses incorporate physician-prescribed interventions. nursing diagnoses incorporate physiologic complications that nurses monitor to detect change in status. nurses manage collaborative problems using physician-prescribed interventions.

nurses manage collaborative problems using physician-prescribed interventions.

Which of the following positions should be avoided in severe back pain? Prone Supine Lateral recumbent Head and thorax elevated 30 degrees

prone

Twenty-four hours after undergoing kidney transplantation, a client develops a hyperacute rejection. To correct this problem, the nurse should prepare the client for: removal of the transplanted kidney. high-dose I.V. cyclosporine (Sandimmune) therapy. bone marrow transplant. intra-abdominal instillation of methylprednisolone sodium succinate (Solu-Medrol).

removal of the transplanted kidney.

A client is admitted for treatment of chronic renal failure (CRF). The nurse knows that this disorder increases the client's risk of: water and sodium retention secondary to a severe decrease in the glomerular filtration rate. a decreased serum phosphate level secondary to kidney failure. an increased serum calcium level secondary to kidney failure. metabolic alkalosis secondary to retention of hydrogen ions.

water and sodium retention secondary to a severe decrease in the glomerular filtration rate.

How should the nurse document this assessment finding? "Biliary cirrhosis noted". "Caput Medusae evident on abdomen" "Hepatomegally palpable in left upper quadrant" "Epistaxis noted bilaterally"

"Caput Medusae evident on abdomen"

After teaching a client with immunodeficiency about ways to prevent infection, the nurse determines that teaching was successful when the client states which of the following? "I should avoid eating cooked fruits and vegetables." "Alcohol is good to clean any skin areas that are dry or chafed." "I should avoid being around other people who have an infection." "I will clean my kitchen counter with hot water."

"I should avoid being around other people who have an infection."

After surgery for removal of cataract, a client is being discharged, and the nurse has completed discharge instruction. Which client statement indicates that the outcome of the teaching plan has been met? "I need to wear sunglasses for the first 3 to 4 days even when I'm inside." "Dots or flashing lights in my vision are to be expected for the first few days." " I should avoid pulling or pushing any object that weighs more than 15 lbs." "I need to keep the eye patch on for about a week after surgery."

"I should avoid pulling or pushing any object that weighs more than 15 lbs."

A nurse is teaching a female client with a history of multiple urinary tract infections (UTIs). Which statement indicates the client understands the teaching about preventing UTIs? "I should wipe from back to front." "I should take a tub bath at least 3 times per week." "I should take at least 1,000 mg of vitamin C each day." "I should limit my fluid intake to limit my trips to the bathroom."

"I should take at least 1,000 mg of vitamin C each day."

A nurse is performing discharge teaching with a client who had a total gastrectomy. Which statement indicates the need for further teaching? "I'm going to visit my pastor weekly for a while." "I will have to take vitamin B12 shots up to 1 year after surgery." "I will call my physician if I begin to have abdominal pain." "I will weight myself each day and record the weight."

"I will have to take vitamin B12 shots up to 1 year after surgery."

The nurse has been educating a patient newly diagnosed with MS. Which of the following statements by the patient indicates an understanding of the education? "I will take hot tub baths to decrease spasms." "I should participate in non-weight-bearing exercises." "I will stretch daily as directed by the physical therapist." "The exercises should be completed quickly to reduce fatigue."

"I will stretch daily as directed by the physical therapist."

A client, who wishes to preserve childbearing, asks the nurse to explain how taking oral contraceptives will work in the management of endometriosis. Which is the best response by the nurse? "Symptoms of endometriosis are increased during normal menstrual cycle." "Contraceptives will allow blood to be diverted to the peritoneal cavity." "Trapping blood causes less pain or discomfort for clients with endometriosis." "Endometriosis is usually cured with surgical menopause."

"Symptoms of endometriosis are increased during normal menstrual cycle."

A client is receiving immunotherapy as part of the treatment plan for an allergic disorder. After administering the therapy, the client states, "I guess I can go home now." Which response by the nurse would be most appropriate? "We need to schedule your next appointment first and then you can leave." "You need to stay about another half-hour so we can make sure you don't have a reaction." "It's okay to leave but make sure to call us if you start to feel strange after an hour or so." "You must stay here so that you can get another injection of a different substance to which you're allergic."

"You need to stay about another half-hour so we can make sure you don't have a reaction."

The nurse is teaching a patient about her rheumatic disease. What statement best helps to explain "autoimmunity"? "You have inherited your parent's immunity to the disease." "Your symptoms are a result of your body attacking itself." "You have antigens to the disease, but it they do not prevent the disease." "You are not immune to the disease causing the symptoms."

"Your symptoms are a result of your body attacking itself."

A patient diagnosed with hypernatremia needs IV sodium replacement. Select the solution that is considered the safest to administer. 0.45% sodium chloride 0.9% sodium chloride 5% dextrose in water 5% dextrose in normal saline solution

0.45% sodium chloride

A client admitted with meningitis is to receive Vancocin (vancomycin) 250 mg in 100 mL intravenously over 60 minutes twice a day. The IV tubing set is calibrated at 15 drops per/mL. At how many drops per minute will the nurse run this solution? Enter the correct number ONLY.

25

The accompanying illustration depicts the neural pathway for vision. Which of the numbered areas is responsible for receiving the stimulus from the retina? 1 Optic Nerve 2 Optic Chiasm 3 Optic Track 4 Visual Cortex

4 The stimulus from the retina is sent to the optic nerve (option D) and then through the optic chiasm (option C), to the optic tract (option B), eventually reaching the visual cortex in the occipital lobe of the brain.

The nurse is teaching the client who will undergo surgery for the creation of a nephrostomy. Which of the images best depicts this type of cutaneous urinary diversion?

A cutaneous diversion involves the creation of an opening through the abdominal wall and skin to allow urine to drain. A nephrosostomy (Option D) allows urine to drain directly from the kidney through a percutaneous catheter through an opening in the flank. An ileal conduit (Option A) is the most common cutaneous diversion, whereby both ureters empty into an isolated section of the ileum. One end of the isolated segment is brought through the abdominal wall and allows urine to drain through a stoma. With a cutaneous ureterostomy (Option B), the ureter is detached from the bladder and brought through the abdominal wall and attached to an opening in the skin. The bladder is sutured to the abdominal wall and a stoma is created through the abdominal and bladder walls for drainage of urine in a vesicostomy (Option C).

Which assessment finding indicates an increased risk of skin cancer? A deep sunburn A dark mole on the client's back An irregular scar on the client's abdomen White irregular patches on the client's arm

A deep sunburn

A patient's colorectal cancer has necessitated a hemicolectomy with the creation of a colostomy. In the 4 days since the surgery, the patient has been unwilling to look at the ostomy or participate in any aspects of ostomy care. What is the nurse's most appropriate response to this observation? Ensure that the patient knows that he or she will be responsible for care after discharge. Reassure the patient that many people are fearful after the creation of an ostomy. Acknowledge the patient's reluctance and initiate discussion of the factors underlying it. Arrange for the patient to be seen by a social worker or spiritual advisor.

Acknowledge the patient's reluctance and initiate discussion of the factors underlying it.

A patient is complaining of ringing in the left ear and hearing loss in the same ear, but does not have any associated dizziness or vertigo. What should this patient be assessed for? Otitis media Acoustic neuroma Labyrinthitis Tinnitus

Acoustic neuroma

A client comes to the emergency department complaining of severe pain in the right flank, nausea, and vomiting. The physician tentatively diagnoses right ureterolithiasis (renal calculi). When planning this client's care, the nurse should assign the highest priority to which nursing diagnosis? Acute pain Risk for infection Impaired urinary elimination Imbalanced nutrition: Less than body requirements

Acute Pain

A client is given a dose of ketorolac (Toradol), a nonsteroidal anti-inflammatory drug for complaints of abdominal pain. Ten minutes after receiving the medication, the client's eyes, lips, and face begin to swell, and the nurse hears stridor. What priority measure should the nurse prepare to do? Intubate the client. Perform an electrocardiogram (ECG). Assess the client's vital signs. Administer epinephrine.

Administer epinephrine.

A patient with end-stage liver disease has developed hypervolemia. What nursing interventions would be most appropriate when addressing the patient's fluid volume excess? Select all that apply. Administering diuretics Administering calcium channel blockers Implementing fluid restrictions Implementing a 1500 kcal/day restriction Enhancing patient positioning

Administering diuretics Implementing fluid restrictions Enhancing patient positioning

A local public health nurse is informed that a cook in a local restaurant has been diagnosed with hepatitis A. What should the nurse advise individuals to obtain who ate at this restaurant and have never received the hepatitis A vaccine? The hepatitis A vaccine Albumin infusion The hepatitis A and B vaccines An immune globulin injection

An immune globulin injection

A nurse is caring for a client who is exhibiting signs and symptoms characteristic of a myocardial infarction (MI). Which statement describes priorities the nurse should establish while performing the physical assessment? Assess the client's level of pain and administer prescribed analgesics. Assess the client's level of anxiety and provide emotional support. Prepare the client for pulmonary artery catheterization. Ensure that the client's family is kept informed of his status.

Assess the client's level of pain and administer prescribed analgesics.

A nurse is performing a home visit to a patient who is recovering following a long course of inpatient treatment for burn injuries. When performing this home visit, the nurse should do which of the following? Assess the patient for signs of electrolyte imbalances. Administer fluids as ordered. Assess the risk for injury recurrence. Assess the patient's psychosocial state.

Assess the patient's psychosocial state.

Which step of the nursing process entails analyzing data related to the patient's health status? Implementation Diagnosis Assessment Evaluation

Assessment

Which of the following would the nurse expect to assess in a client with hepatic encephalopathy? Increased motor activity Asterixis Negative Babinski reflex Irritability

Asterixis

How would you interpret this EKG rhythm and what nursing interventions would you anticipate? Atrial Flutter, administer ordered coumadin and assess for stroke symptoms. First degree heart block, assess for syncope and anticipate pace maker insertion. Ventricular Tachycardia, initiate CPR Pre-mature ventricular contractions, assess for symptoms, administer ordered lidocaine if symptomatic

Atrial Flutter, administer ordered coumadin and assess for stroke symptoms.

A client has been having joint pain and swelling in the left foot and is diagnosed with rheumatoid arthritis. The symptoms began suddenly without any identifiable cause, and the client has significant joint destruction. What type of disease is this considered? An exacerbation of a previous disorder Autoimmune An alloimmunity disorder A cause-and-effect relationship

Autoimmune

The client has been brought to the Emergency Department by their caregiver. The caregiver says that they found the client diaphoretic, nauseated, flushed and complaining of a pounding headache when they came on shift. What are these symptoms indicative of? Concussion Autonomic dysreflexia Spinal shock Contusion

Autonomic dysreflexia

A client is taking a corticosteroid for the treatment of systemic lupus erythematosus. When the nurse is providing instructions about the medication to the client, what priority information should be included? If the client experiences nausea, omit the dose. The client should be alert for joint aches. This medication is commonly used for many inflammatory reactions and is relatively safe. Be alert for signs and symptoms of infection and report them immediately to the physician.

Be alert for signs and symptoms of infection and report them immediately to the physician.

Which of the following is considered a stimulant laxative? Magnesium hydroxide (milk of Magnesia) Bisacodyl (Dulcolax) Mineral oil Psyllium hydrophilic mucilloid (Metamucil)

Bisacodyl (Dulcolax)

A client with chronic renal failure (CRF) is admitted to the urology unit. Which diagnostic test results are consistent with CRF? Increased pH with decreased hydrogen ions Increased serum levels of potassium, magnesium, and calcium Blood urea nitrogen (BUN) 100 mg/dl and serum creatinine 6.5 mg/dl Uric acid analysis 3.5 mg/dl and phenolsulfonphthalein (PSP) excretion 75%

Blood urea nitrogen (BUN) 100 mg/dl and serum creatinine 6.5 mg/dl

Based on her knowledge of the most common site of metastatic spread for breast cancer, the nurse would recommend periodic assessments of the: Bone Brain Lungs Liver

Bone

The nurse recognizes that the client with osteomyelitis is at risk for: Impingement syndrome Metastatic bone disease Bone abscess formation Pathological fractures

Bone abscess formation

Which of the following is an inaccurate clinical manifestation associated with hemorrhage? Tachycardia Bradycardia Tachypnea Hypotension

Bradycardia

A client is in the emergency department following a fall on the face. The client reports facial pain. The nurse assesses bleeding from nasal cuts and from the nares, a deformity to the nose, periorbital ecchymoses, and some clear fluid draining from the right nostril. The first action of the nurse is to Apply an ice pack to the nose. Reassure the client that the nose is not fractured. Administer prescribed oral ibuprofen (Motrin). Check the clear fluid for glucose.

Check the clear fluid for glucose.

Which of the following terms is used to describe rapid, jerky, involuntary, and purposeless movements of the extremities? Bradykinesia Dyskinesia Chorea Spondylosis

Chorea

A public health nurse is preparing an educational campaign to address a recent local increase in the incidence of HIV infection. The nurse should prioritize which of the following interventions? Lifestyle actions that improve immune function Educational programs that focus on control and prevention Appropriate use of standard precautions Screening programs for youth and young adults

Educational programs that focus on control and prevention

The nurse identifies a nursing diagnosis of imbalanced nutrition, less than body requirements related to difficulty in chewing and swallowing for a client with Parkinson's disease. Which of the following would be most appropriate for the nurse to integrate into the client's plan of care? Raise the head of the client's bed about 30 degrees during meals. Encourage the use of liquids that are thin in consistency. Arrange for specialized utensils for the client to use when eating. Encourage the client to massage the facial and neck muscles before eating.

Encourage the client to massage the facial and neck muscles before eating.

A client hospitalized with pneumonia has thick, tenacious secretions. Which intervention should the nurse include when planning this client's care? Turning the client every 2 hours Elevating the head of the bed 30 degrees Encouraging increased fluid intake Maintaining a cool room temperature

Encouraging increased fluid intake

The nurse is planning an education program for women of childbearing years. The nurse recognizes that primary prevention of osteoporosis includes: Engaging in non-weight-bearing exercises daily Ensuring adequate calcium and vitamin D intake Undergoing assessment of serum calcium levels every year Having a DXA beginning at age 35 years

Ensuring adequate calcium and vitamin D intake

A surgeon is discussing surgery with a client diagnosed with colon cancer. The client is visibly shaken over the possibility of a colostomy. Based on the client's response, the surgeon should collaborate with which health team member? Social worker Staff nurse Clinical educator Enterostomal nurse

Enterostomal nurse

A patient is brought by ambulance to the ED after suffering what the family thinks is a stroke. The nurse caring for this patient is aware that an absolute contraindication for thrombolytic therapy is what? Evidence of hemorrhagic stroke Blood pressure of ≥ 180/110 mm Hg Evidence of stroke evolution Previous thrombolytic therapy within the past 12 months

Evidence of hemorrhagic stroke

During an interview for an ambulatory clinic position, the nurse notices that family planning counseling is included in the job description. Being a devout Catholic, how should the nurse proceed with the interview? Excuse herself from the interview stating she is Catholic Continue the interview; other nurses at the center can provide family counseling Realize the ethical obligation to provide care to all faiths, and continue the interview process Continue the interview and only provide patients with information related to abstinence

Excuse herself from the interview stating she is Catholic

A client is recovering from gastric surgery. Which of the following is the correct position for the nurse to place this client? Supine Semi-Fowler's Trendlenberg Fowler's

Fowler's

Bill Jenkins has suffered from a burn on his leg related to an engine fire. Burn depth is determined by assessing the color, characteristics of the skin, and sensation in the area. When the burn area was assessed, it was determined that he felt no pain in the area and that it appeared charred. What depth of burn injury would he be said to have? Full thickness (third degree) Superficial (first degree) Superficial partial-thickness and deep partial-thickness (second degree) Fourth degree

Full thickness (third degree)

A client with a burn injury is in acute stress. Which of the following complications is prone to develop in this client? Anemia Gastric ulcers Hyperthyroidism Cardiac arrest

Gastric ulcers

A 30-year-old client whose mother died of breast cancer at age 44 and whose sister has ovarian cancer, is concerned about developing cancer. As a member of the oncology multidisciplinary team, the nurse should suggest that the client ask the physician about which topic? Mammogram Papanicolaou (Pap) testing every 6 months Contacting the American Cancer Society Genetic counseling

Genetic counseling

During assessment of a patient who has been taking Dilantin for seizure management for 3 years, the nurse notices one of the side effects that should be reported. What is that side effect? Alopecia Gingival hyperplasia Diplopia Ataxia

Gingival hyperplasia

A client with esophageal cancer has difficulty in swallowing. Which of the following would be appropriate to help the client achieve improved nutrition? Give high-protein, semiliquid foods Provide oral liquids through a straw Give liquid supplements for meals Encourage small, frequent meals

Give high-protein, semiliquid foods

A nurse is assisting with the clinical examination for determination of brain death for a client, related to potential organ donation. All 50 states in the United States recognize uniform criteria for brain death. The nurse is aware that the three cardinal signs of brain death on clinical examination are all of the following except: Coma Absence of brain stem reflexes Apnea Glasgow Coma Scale of 6

Glasgow Coma Scale of 6

A nurse is reviewing dietary guidelines with a client who recently had Roux-en-Y gastric bypass (RYGB) surgery. Which of the following would be included? Select all that apply. Gradually progress to five or six small meals daily, with each feeding providing protein, fat, and complex carbohydrate. Restrict total amount to less than one cup. Avoid all sweets. Sip on clear liquids with meals Encourage consumption of foods such as doughy breads, pasta, rice, skins and seeds of fruits and vegetables, nuts, and popcorn.

Gradually progress to five or six small meals daily, with each feeding providing protein, fat, and complex carbohydrate. Restrict total amount to less than one cup. Avoid all sweets.

What should a male client older than age 50 do to help ensure early identification of prostate cancer? Have a digital rectal examination and prostate-specific antigen (PSA) test done yearly. Have a transrectal ultrasound every 5 years. Perform monthly testicular self-examinations, especially after age 50. Have a complete blood count (CBC) and blood urea nitrogen (BUN) and creatinine levels checked yearly.

Have a digital rectal examination and prostate-specific antigen (PSA) test done yearly.

A patient is admitted to the burn unit after being transported from a facility 1000 miles away. The patient has burns to the groin area and circumferential burns to both upper thighs. When assessing the patient's legs distal to the wound site, the nurse should be cognizant of the risk of what complication? Ischemia Referred pain Cellulitis Venous thromboembolism (VTE)

Ischemia

A patient is diagnosed with the most common type of uterine fibroid, an intramural fibroid. The nurse includes which of the following information in teaching the patient about this type of fibroid? It lies underneath the outermost layer of the uterus. It grows within the wall of the uterine muscle. It grows below the inner uterine surface. It arises from inside or outside the surface of the uterine muscle.

It grows within the wall of the uterine muscle.

Which of the following is a true statement regarding psoriasis? It is a chronic, infectious inflammatory disease. It is characterized by patches of redness covered with silvery scales. A cure is possible with prompt treatment. The onset typically occurs in young children.

It is characterized by patches of redness covered with silvery scales.

Which of the following is accurate regarding sildenafil (Viagra)? It can be taken twice daily for increased effect. The medication should be taken right before intercourse. Its side effects include headache, flushing, and dizziness. There does not need to be sexual stimulation to produce an erection.

Its side effects include headache, flushing, and dizziness.

While conducting a physical examination of a client, which of the following skin findings would alert the nurse to the possibility of liver problems? Select all that apply. Jaundice Petechiae Ecchymoses Cyanosis of the lips Aphthous stomatitis

Jaundice Petechiae Ecchymoses

Nursing students are reviewing information about the different manifestations associated with AIDS. The students demonstrate understanding of these manifestations when they identify which of the following as the most common HIV-related malignancy? Invasive cervical cancer Non-Hodgkin's lymphoma B-cell lymphoma Kaposi's sarcoma

Kaposi's sarcoma

The nurse assesses the patient and observes reddish-purple to dark blue macules, plaques and nodules. The nurse recognizes that these manifestations are associated with which of the following conditions? Platelet disorders Kaposi's sarcoma Allergic reactions Syphilis

Kaposi's sarcoma

Which of the following enzymes aids in the digestion of fats? Lipase Amylase Secretin Trypsin

Lipase

A 66-year-old African-American client has recently visited a physician to confirm a diagnosis of gastric cancer. The client has a history of tobacco use and was diagnosed 10 years ago with pernicious anemia. He and his family are shocked about the possibility of cancer because he was asymptomatic prior to recent complaints of pain and multiple gastrointestinal symptoms. On the basis of knowledge of disease progression, the nurse assumes that organs adjacent to the stomach are also affected. Which of the following organs may be affected? Choose all that apply. Liver Pancreas Bladder Duodenum Lungs

Liver Pancreas Duodenum

After teaching a group of students about irritable bowel syndrome and antidiarrheal agents, the instructor determines that the teaching was effective when the students identify which of the following as an example of an opiate-related antidiarrheal agent? Loperamide (Imodium) Bismuth subsalicylate (Pepto-Bismol) Kaolin and pectin (Kaopectate) Bisacodyl (Dulcolax)

Loperamide (Imodium)

A nurse conducted a history and physical for a newly admitted patient who states, "My arms are too short. I have to hold my book at a distance to read." The nurse knows that the patient is most likely experiencing: Opacity in the lens. Loss of accommodative power in the lens. Shrinkage of the vitreous body. Decreased eye muscle tone.

Loss of accommodative power in the lens.

Which of the following is the recommended dietary treatment for a client with chronic cholecystitis? Low-fat diet High-fiber diet Low-residue diet Low-protein diet

Low-fat diet

From the following profiles of clients, which client would be most likely to undergo the diagnostic test of cholecystography? Steven, suspected of having a tumor in the colon Andrew, suspected of having esophageal abnormalities Mark, suspected of having stones in the gallbladder Sandra, suspected of having lesions in the liver

Mark, suspected of having stones in the gallbladder

Rebleeding may occur from a peptic ulcer and often warrants surgical interventions. Signs of bleeding include which of the following? Mental confusion Bradycardia Bradypnea Hypertension

Mental confusion

While examining a client's leg, a nurse notes an open ulceration with visible granulation tissue in the wound. Until a wound specialist can be contacted, which type of dressing should the nurse apply? Dry sterile dressing Sterile petroleum gauze Moist sterile saline gauze Povidone-iodine-soaked gauze

Moist sterile saline gauze

The nurse is caring for a patient who is postoperative following a craniotomy. When writing the plan of care, the nurse identifies a diagnosis of "deficient fluid volume related to fluid restriction and osmotic diuretic use." What would be an appropriate intervention for this diagnosis? Change the patient's position as indicated. Monitor serum electrolytes. Maintain NPO status. Monitor arterial blood gas (ABG) values.

Monitor serum electrolytes.

A client with decreased urine output refractory to fluid challenges is evaluated for renal failure. Which condition may cause the intrinsic (intrarenal) form of acute renal failure? Poor perfusion to the kidneys Damage to cells in the adrenal cortex Obstruction of the urinary collecting system Nephrotoxic injury secondary to use of contrast media

Nephrotoxic injury secondary to use of contrast media

A client is brought to the emergency department in a confused state, with slurred speech, characteristics of a headache, and right facial droop. The vital signs reveal a blood pressure of 170/88 mm Hg, pulse of 92 beats/minute, and respirations at 24 breaths/minute. On which bodily system does the nurse focus the nursing assessment? Cardiovascular system Respiratory system Endocrine system Neurovascular system

Neurovascular system

A nursing student notes that a client has a new physician order for a proton-pump inhibitor. The student knows that a frequently prescribed proton-pump inhibitor of gastric acid is Nexium Pepcid Tagamet Zantac

Nexium

Which of the following would be included as a postoperative intervention for the patient undergoing a laparoscopic cholecystectomy? NPO status postop for 2 days Low-carbohydrate, low-protein diet immediately after surgery Semi-Fowler's position Observe color of sclera

Observe color of sclera

The registered nurse (RN) is responsible for delegating patient care responsibilities to licensed practical nurses (LPNs) as well as ancillary personnel. What would be the most appropriate task to delegate to a nursing assistant? Assessing the degree of lower leg edema in a patient on bed rest Obtaining vital signs for a patient that has been hospitalized for 3 days Measuring the circumference of a patient's calf for edema Recording the size and appearance of a decubitus ulcer

Obtaining vital signs for a patient that has been hospitalized for 3 days

A patient has undergone a liver biopsy. Following the procedure, the nurse should place the patient in which of the following positions? On the left side Trendelenburg On the right side High Fowler's

On the right side

The nurse is caring for a patient in the ICU who has a brain stem herniation and who is exhibiting an altered level of consciousness. Monitoring reveals that the patient's mean arterial pressure (MAP) is 60 mm Hg with an intracranial pressure (ICP) reading of 5 mm Hg. What is the nurse's most appropriate action? Position the patient in the high Fowler's position as tolerated. Administer osmotic diuretics as ordered. Participate in interventions to increase cerebral perfusion pressure. Prepare the patient for craniotomy.

Participate in interventions to increase cerebral perfusion pressure.

A client with a T4-level spinal cord injury (SCI) reports severe headache. The nurse notes profuse diaphoresis of the client's forehead and scalp and suspects autonomic dysreflexia. What is the first thing the nurse will do? Place the client in a sitting position. Lay the client flat. Apply antiembolic stockings. Notify the physician.

Place the client in a sitting position.

The client has returned to the floor following a radical neck dissection. Anesthesia has worn off. It is most important for the nurse to Place the client in the Fowler's position. Administer morphine for report of pain. Provide feeding through the gastrostomy tube. Empty the Jackson-Pratt device (portable drainage device).

Place the client in the Fowler's position.

Which of the following is the overall aim of glaucoma treatment? Prevent optic nerve damage Optimize the patient's remaining vision Reverse optic nerve damage Reattach the retina

Prevent optic nerve damage

A client is admitted to the health care facility with abdominal pain, a low-grade fever, abdominal distention, and weight loss. The physician diagnoses acute pancreatitis. What is the primary goal of nursing care for this client? Relieving abdominal pain Preventing fluid volume overload Maintaining adequate nutritional status Teaching about the disease and its treatment

Relieving abdominal pain

A client being treated for pancreatitis faces the risk of atelectasis. Which of the following interventions would be important to implement to minimize this risk? Monitor pulse oximetry every hour. Withhold oral feedings for the client. Instruct the client to avoid coughing. Reposition the client every 2 hours.

Reposition the client every 2 hours.

The nurse on a urology unit is working with a patient who has been diagnosed with oxalate renal calculi. When planning this patient's health education, what nutritional guidelines should the nurse provide? Restrict protein intake as ordered. Increase intake of potassium-rich foods. Follow a low-calcium diet. Encourage intake of food containing oxalates.

Restrict protein intake as ordered.

The nurse is participating in a health fair about fire safety. Which of the following interventions helps to minimize the risk of further injury to an affected person at a scene of a fire when clothes catch fire? Roll the patient in a blanket. Cover the patient with a wet cloth. Place the patient with the head positioned slightly below the rest of the body. Avoid immediate IV fluid therapy.

Roll the patient in a blanket.

A male client comes to the walk-in clinic stating he has just found out he has been exposed to HIV. The client tests HIV-negative after an enzyme-linked immunosorbent assay test. The client wants to know the precise sources through which the HIV infection is transmitted, as he wants to take proper precautions. Which of the following body fluids transmits the HIV infection? Sweat Saliva Urine Semen

Semen

During his annual physical examination, a retired airplane mechanic reports noticeable hearing loss. The nurse practitioner prescribes a series of hearing tests to confirm or rule out noise-induced hearing loss, which is classified as a: Conduction problem. Sensorineural loss. Mixed cause. Psychogenic issue.

Sensorineural loss.

Laboratory studies indicate a client's blood glucose level is 185 mg/dl. Two hours have passed since the client ate breakfast. Which test would yield the most conclusive diagnostic information about the client's glucose use? Fasting blood glucose test 6-hour glucose tolerance test Serum glycosylated hemoglobin (Hb A1c) Urine ketones

Serum glycosylated hemoglobin (Hb A1c)

A 35-year-old female client is requesting information about mammograms and breast cancer. She isn't considered at high risk for breast cancer. What should the nurse tell this client? She should have had a baseline mammogram before age 30. She should eat a low-fat diet to further decrease her risk of breast cancer. She should perform breast self-examination during the first 5 days of each menstrual cycle. When she begins having yearly mammograms, breast self-examinations will no longer be necessary.

She should eat a low-fat diet to further decrease her risk of breast cancer.

A client suspected of having colorectal cancer requires which diagnostic study to confirm the diagnosis? Stool Hematest Carcinoembryonic antigen (CEA) Sigmoidoscopy Abdominal computed tomography (CT) scan

Sigmoidoscopy

A nurse is conducting a detailed skin assessment on an 80-year-old client. Which finding requires further investigation? Yellow, waxy deposits on the lower eyelids Bright red moles on the hands Several areas of dry, scaly skin Small, waxy nodule with pearly borders

Small, waxy nodule with pearly borders

Atrial fibrillation is common in patients with atrial septal defects (ASDs) and further increases the risk of which of the following? Stroke Cardiomegaly Heart failure Splenomegaly

Stroke

A patient who experienced an upper GI bleed due to gastritis has had the bleeding controlled and the patient's condition is now stable. For the next several hours, the nurse caring for this patient should assess for what signs and symptoms of recurrence? Tachycardia, hypotension, and tachypnea Tarry, foul-smelling stools Diaphoresis and sudden onset of abdominal pain Sudden thirst, unrelieved by oral fluid administration

Tachycardia, hypotension, and tachypnea

A mother brings her young child to the clinic for an evaluation of an infection. The mother states, "He's been taking antibiotics now for more than 2 months and still doesn't seem any better. It's like he's always sick." During the history and physical examination, which of the following would alert the nurse to suspect a primary immunodeficiency? Superficial wound on the child's left leg Ten ear infections in the past year Weight within age-appropriate parameters History of fungal diaper rash

Ten ear infections in the past year

A nurse is caring for a client who's ordered continuous ambulatory peritoneal dialysis (CAPD). Which finding should lead the nurse to question the client's suitability for CAPD? The client is blind in his right eye. The client has a history of severe anemia during hemodialysis. The client has a history of diverticulitis. The client is on the kidney transplant waiting list.

The client has a history of diverticulitis.

After having transurethral resection of the prostate (TURP), a client returns to the unit with a three-way indwelling urinary catheter and continuous closed bladder irrigation. Which finding suggests that the client's catheter is occluded? The urine in the drainage bag appears red to pink. The client reports bladder spasms and the urge to void. The normal saline irrigant is infusing at a rate of 50 drops/minute. About 1,000 ml of irrigant have been instilled; 1,200 ml of drainage have been returned.

The client reports bladder spasms and the urge to void.

The Monro-Kellie hypothesis refers to which of the following statements? The dynamic equilibrium of cranial contents. The brain is unresponsive to the environment. The brain's attempt to restore blood flow by increasing arterial pressure to overcome the increased intracranial pressure. The patient is wakeful but devoid of conscious content, without cognitive or affective mental function.

The dynamic equilibrium of cranial contents.

A patient's screening colonoscopy revealed the presence of numerous polyps in the large bowel. What principle should guide the subsequent treatment of this patient's health problem? Adherence to a high-fiber diet will help the polyps resolve. The patient should be assured that these are a normal, age-related physiologic change. The patient's polyps constitute a risk factor for cancer. The presence of polyps is associated with an increased risk of bowel obstruction.

The patient's polyps constitute a risk factor for cancer.

A patient's screening colonoscopy revealed the presence of numerous polyps in the large bowel. What principle should guide the subsequent treatment of this patient's health problem? Adherence to a high-fiber diet will help the polyps resolve. The patient should be assured that these are a normal, age-related physiologic change. The patient's polyps constitute a risk factor for cancer. The presence of polyps is associated with an increased risk of bowel obstruction.

The patient's polyps constitute a risk factor for cancer.

A nurse chooses a quiet, private area to conduct an end-of-shift report to the oncoming nurse. Following this procedure is necessary because of what ethical problem in nursing? Maintaining trust between nurse and nurse is necessary for proper client care. The right of confidentiality is essential to protect each client's private information. Respect for clients ensures that nurses treat them in such a way that enables clients to make choices. The ANA's Code of Ethics for Nurses states that all nurses must make an effort to report off at the end of a shift.

The right of confidentiality is essential to protect each client's private information.

An older adult patient informs the nurse, "I don't see as well as I used to." What should the nurse explain to the patient about why vision becomes less efficient with age? (Select all the apply.) There is a decrease in pupil size. There is slowing of accommodation. There is an increase in lens opaqueness. Most older patients develop glaucoma. The optic nerve begins to degenerate.

There is a decrease in pupil size. There is slowing of accommodation. There is an increase in lens opaqueness.

Total parental nutrition (TPN) should be used cautiously in patients with pancreatitis due to which of the following? They are at risk for gallbladder contraction. They are at risk for hepatic encephalopathy. They can digest high-fat foods. They cannot tolerate high-glucose concentration.

They cannot tolerate high-glucose concentration.

An elderly woman found with a head injury on the floor of her home is subsequently admitted to the neurologic ICU. What is the best rationale for the following physician orders: elevate the HOB; keep the head in neutral alignment with no neck flexion or head rotation; avoid sharp hip flexion? To decrease cerebral arterial pressure To avoid impeding venous outflow To prevent flexion contractures To prevent aspiration of stomach contents

To avoid impeding venous outflow

Joe Blake, age 16 years, was playing soccer when he received an injury to the ligaments of his knee. He was brought to the emergency department because of pain, joint instability, and difficulty walking. His injury was judged not to require surgery. Which of the following interventions would not be included in his care? Traction Joint immobilization Limited weight bearing Ice and NSAIDs

Traction

Joe Blake, age 16 years, was playing soccer when he received an injury to the ligaments of his knee. He was brought to the emergency department because of pain, joint instability, and difficulty walking. His injury was judged not to require surgery. Which of the following interventions would not be included in his care? Traction Joint immobilization Limited weight bearing Ice and NSAIDs

Traction

Which of the following would be most important to include in a teaching plan for a client who has had a vasectomy? Using a reliable method of contraception for several weeks. Applying warm compresses to the scrotum for the first 24 hours Taking a prescribed opioid analgesic for pain relief Resuming sexual activity in 24 to 48 hours

Using a reliable method of contraception for several weeks.

A nurse visits the employee health department because of mild itching and a rash on both hands. During the assessment interview, the employee health nurse should focus on: medication allergies. life stressors the nurse may be experiencing. chemical and latex glove use. laundry detergent or bath soap changes.

chemical and latex glove use.

Which of the following insults or abnormalities can cause an ischemic stroke? Cocaine use Arteriovenous malformation Trauma Intracerebral aneurysm rupture

cocaine use

Audiometry confirms a client's chronic progressive hearing loss. Further investigation reveals ankylosis of the stapes in the oval window, a condition that prevents sound transmission. This type of hearing loss is called: functional hearing loss. fluctuating hearing loss. sensorineural hearing loss. conductive hearing loss.

conductive hearing loss.

Medical management of arthropod-borne virus (arboviral) encephalitis is aimed at preventing renal insufficiency. controlling seizures and increased intracranial pressure. maintaining hemodynamic stability and adequate cardiac output. preventing muscular atrophy.

controlling seizures and increased intracranial pressure

A client asks the nurse what PSA is. The nurse should reply that it stands for: prostate-specific antigen, which is used to screen for prostate cancer. protein serum antigen, which is used to determine protein levels. pneumococcal strep antigen, which is a bacteria that causes pneumonia. Papanicolaou-specific antigen, which is used to screen for cervical cancer.

prostate-specific antigen, which is used to screen for prostate cancer.

A client with chronic renal failure (CRF) is receiving a hemodialysis treatment. After hemodialysis, the nurse knows that the client is most likely to experience: hematuria. weight loss. increased urine output. increased blood pressure.

weight loss

A client with cholelithiasis has a gallstone lodged in the common bile duct. When assessing this client, the nurse expects to note: yellow sclerae. light amber urine. circumoral pallor. black, tarry stools.

yellow sclerae.


Conjuntos de estudio relacionados

Abeka Grade 4 - History of Our United States -Chapter 2: The Years of Conquest (Chapter Checkup)

View Set

Saunders: Developmental Stages: End of Life

View Set

Chapter 20: The Progressive Era, 1890-1920

View Set

OMNI Air International City Codes

View Set

Chapter 65 med surg (Assessment of function worksheet)

View Set